if you roll two fair six-sided dice, what is the probability that the sum is 4 44 or higher?

Answers

Answer 1
444 would be the answer.
Answer 2

The probability of rolling two fair six-sided dice and getting a sum of 4 or higher is 11/12

To calculate the probability of rolling two fair six-sided dice and getting a sum of 4 or higher, we first need to calculate the total number of possible outcomes.

The number of possible outcomes when rolling two dice is 6 × 6 = 36, since each die has 6 possible outcomes.

Now, let's find the number of outcomes that result in a sum of 4 or higher. We can do this by listing all the possible outcomes:

Sum of 4: (1, 3), (2, 2), (3, 1) = 3 outcomes

Sum of 5: (1, 4), (2, 3), (3, 2), (4, 1) = 4 outcomes

Sum of 6: (1, 5), (2, 4), (3, 3), (4, 2), (5, 1) = 5 outcomes

Sum of 7: (1, 6), (2, 5), (3, 4), (4, 3), (5, 2), (6, 1) = 6 outcomes

Sum of 8: (2, 6), (3, 5), (4, 4), (5, 3), (6, 2) = 5 outcomes

Sum of 9: (3, 6), (4, 5), (5, 4), (6, 3) = 4 outcomes

Sum of 10: (4, 6), (5, 5), (6, 4) = 3 outcomes

Sum of 11: (5, 6), (6, 5) = 2 outcomes

Sum of 12: (6, 6) = 1 outcome

Therefore, the number of outcomes that result in a sum of 4 or higher is 3 + 4 + 5 + 6 + 5 + 4 + 3 + 2 + 1 = 33.

Therefore, the probability of rolling two fair six-sided dice and getting a sum of 4 or higher is 33/36 = 11/12.

To find the probability of getting a sum of 44 or higher, we need to subtract the probability of getting a sum of 43 or lower from 1:

Sum of 2: (1, 1) = 1 outcome

Sum of 3: (1, 2), (2, 1) = 2 outcomes

Sum of 4: (1, 3), (2, 2), (3, 1) = 3 outcomes

Sum of 5: (1, 4), (2, 3), (3, 2), (4, 1) = 4 outcomes

Sum of 6: (1, 5), (2, 4), (3, 3), (4, 2), (5, 1) = 5 outcomes

Sum of 7: (1, 6), (2, 5), (3, 4), (4, 3), (5, 2), (6, 1) = 6 outcomes

Sum of 8: (2, 6), (3, 5), (4, 4), (5, 3), (6, 2) = 5 outcomes

Sum of 9: (3, 6), (4, 5), (5, 4), (6, 3) = 4 outcomes

To learn more about probability, click here:

https://brainly.com/question/24756209

#SPJ11


Related Questions

Can someone help me please

Answers

Answer:

A, 48 degrees

Step-by-step explanation:

180 - 42 - 90 = 48 (all interior angles add up to 48)

Answer:

it’s A

Step-by-step explanation:

it’s a because the right triangle is always 90°.As you can see there is 42° so you subtract to find the missing number.

from the following quadratic function , g(x)= -4(x+2)^2-3 identify the difference between its parent function f(x)=x^2

Answers

Thus, through the steps of horizontal translation, dilation and at last vertical translation, the new quadratic function , g(x)= -4(x+2)²-3 from the parent function f(x)=x².

Explain about the parent function:

The simplest function which nonetheless complies with a particular type of function's definition is a parent function. For instance, y = x would be the parent function when considering the linear functions that make a family of functions. The most basic linear function is this one.

In addition, by applying various transformations to the graph of the parent function, all of the functions in a family of functions can also be derived from it. Vertical shifts, extending or compressing both horizontally and vertically, reflecting and over x or y axes, and horizontal shifts are some of these transformations.

Given parent function:  f(x)=x²

new quadratic function , g(x)= -4(x+2)²-3

there is the translation of 2 units to right such that 2 is added to x.Now, there is dilation with the scale factor of -4.At last the function is shifted 3 units down

Thus, through the steps of horizontal translation, dilation and at last vertical translation, the new quadratic function , g(x)= -4(x+2)²-3 from the parent function f(x)=x².

Know ore about the parent function

https://brainly.com/question/29233606

#SPJ1

Complete question:

from the following quadratic function , g(x)= -4(x+2)²-3 .identify the difference between its parent function f(x)=x² and  g(x).

Please please please answer this i need ittt

Answers

Answer: 81

Step-by-step explanation: this would be 81 because [tex]3^{3}[/tex] is basically 3 x 3 x 3 and 3 x 3 x 3=27 then multiply 27 by 3, and you get 81

Answer:

3^4. my answer needs to be 20+character sooooooooo

Quality-control research determined that of all new


cars sold by Sherman Motors, 8% will require a


minor repair during the first year of ownership.


Suppose you survey the owners of three cars from


Sherman Motors. Find the probability to the nearest


percent that exactly one car will require a minor


repair in the first year

Answers

The probability that out of three exactly one car will require a minor repair in the first year = 33%

P(E ) = no. of favorable outcome/total no. of outcome

E here represents exactly one car that requires a minor repair.

out of three cars, exactly one car will need a minor repair

No. of favorable outcome = 1

Total no. of outcome = 3

Now, putting value in P(E) we get

P(E) = 1/3

P(E) = 0.333

To get percentage we multiply by 100

P(E) = 0.333 × 100

P(E) = 33.3

The probability to the nearest percent = 33%

To know more about probability click here :

https://brainly.com/question/30034780

#SPJ4

A fair 6-sides die is rolled 550 times. What is a reasonable prediction for the number of times the event of landing on an even number?

Answers

The prediction for the number of times the event of landing on an even number in 550 rolls is 275

Estimating the reasonable prediction

From the question, we have the following parameters that can be used in our computation:

The number of times = 550

The sample space of a fair 6-sided die is

S = {1, 2, 3, 4, 5, 6}

And as such the even numbers are

Even = {2, 4, 6}

This means that in a fair 6-sided die, we have

P(Even) = 3/6

When evaluated, we have

P(Even) = 1/2

So, when the die is rolled 550 times, we have

Expected value = 1/2 * 550

Evaluate

Expected value = 275

Hence, the number of times is 275

Read more about expected value at

brainly.com/question/15858152

#SPJ1

Let G be the center of the equilateral triangle XYZ. A dilation centered at G with scale factor -3/4 is applied to triangle XYZ, to obtain triangle X'Y'Z'. Let A be the area of the region that is contained in both triangles XYZ and X'Y'Z'. Find A/the area of XYZ.

Answers

The calculated value of the expression A/the area of XYZ is [tex]\frac{49\sqrt3}{216}[/tex]

Finding the value of A/the area of XYZ

From the question, we have the following parameters that can be used in our computation:

Center of the equilateral triangle XYZ = GDilation centered at G with scale factor = 3/4

By the ratio of corresponding sides (see attachment for figure), we have

(x + 2y)/(2x + y) = 3/4

By comparison, we have

x + 2y = 3

2x + y = 4

This gives

(x, y) = (5/3, 2/3)

The triangles are equilateral triangles

So, we have

Area of XYZ = 1/2 * side length² * sin(60)

This gives

Area of XYZ = 1/2 * (2x + y)² * sin(60)

Substitute the known values in the above equation

Area of XYZ = 1/2 * (4)² * sin(60)

Evaluate

Area of XYZ = 4√3

The region A is a trapezoid

So, the area is

A = 1/2 * Sum of parallel sides * height

So, we have

A = 1/2 * (x + y) * (x² - y²)

Recall that (x, y) = (5/3, 2/3)

So, we have

A = 1/2 * (5/3 + 2/3) * ((5/3)² - (2/3)²)

Evaluate

A = 49/18

Finding A/the area of XYZ, we have

A/the area of XYZ = 49/18 ÷ 4√3

This gives

A/the area of XYZ = 49/72 ÷ √3

Rationalize

A/the area of XYZ = [tex]\frac{49\sqrt3}{216}[/tex]

Hence, the value of the expression is [tex]\frac{49\sqrt3}{216}[/tex]

Read more about scale factor at

https://brainly.com/question/29229124

#SPJ1

Complete question

Let G be the center of the equilateral triangle XYZ. A dilation centered at G with scale factor -3/4 is applied to triangle XYZ, to obtain triangle X'Y'Z'.  Let A be the area of the region that is contained in both triangles XYZ and X'Y'Z'. Find A/the area of XYZ.

XY = 2x + y

X'Z' = x + 2y

Region A is a trapezoid with parallel sides y & x and height x² - y²

M
what is the rate of return when 30 shares of stock
a. purchased for $20/share, are sold for $720? the
commission on the sale is $6.
rate
return = [?] %
give your answer as a percent rounded to the
nearest tenth.

Answers

The rate of return is 19%, rounded to the nearest tenth.

Given that a purchased for $ 20/share, are sold for $ 720. $ 6 is the  commission on the sale. We need to calculate the total cost of the investment and the total proceeds from the sale, and then use the formula for rate of return.

The total value should be

= 20 × 30

= $ 600

Since, it is sold for $ 720 along with commission of $6 so final money should be  

= 720 - 6

= $ 714.

Now rate of return is

= (714 - 600)/714*100

= 114/600*100

= 19%

Therefore, the rate of return is 19%, rounded to the nearest tenth.

Learn more about Rate of Return here

brainly.com/question/21691170

#SPJ4

The sequence 10, 9. 5, 9. 0, 8. 5,. Has a common difference on


The sequence 200, 100, 50, 25,


has a common ratio of

Answers

The sequence 10, 9. 5, 9. 0, 8. 5,. has a common difference on -0.5

The sequence 200, 100, 50, 25, has a common ratio of 1/2

Let's start by discussing the sequence 10, 9.5, 9.0, 8.5. We can observe that each term is decreasing by 0.5. This means that the sequence has a common difference of -0.5.

In mathematical terms, the common difference is the constant value that is added or subtracted from each term in the sequence to obtain the next term. In this case, we can write the sequence as:

10, 10 - 0.5, 10 - 1.0, 10 - 1.5

where the common difference is -0.5.

Now, let's consider the sequence 200, 100, 50, 25. We can observe that each term is obtained by dividing the previous term by 2. This means that the sequence has a common ratio of 1/2.

In mathematical terms, the common ratio is the constant value that is multiplied by each term in the sequence to obtain the next term. In this case, we can write the sequence as:

200, 200/2, (200/2)/2, ((200/2)/2)/2

where the common ratio is 1/2.

To know more about sequence here

https://brainly.com/question/30262438

#SPJ4

Translate the following statement into a mathematical equation:

Five times a number, minus three, is twelve.

Answers

Its translation is 5×3-3=12

Marsha is considering purchasing 3 points on a $350,000 home mortgage for 20 years. If she
purchases the 3 points, at a cost of 1 percent per point, her monthly mortgage would be
approximately $1,878.63. If she decides not to purchase any points, Mercedes' monthly
payment would be approximately $1,987.13. How much money will Mercedes save over the life
of the loan if she purchases the 3 points?

Answers

Marsha would save $26,040 over the life of the loan if she purchases the 3 points.

First, let's calculate the monthly payment if Marsha doesn't purchase any points. We can use a mortgage calculator or the PMT function in Excel to find;

PMT = $1,987.13

Now, let's calculate the monthly payment if Marsha purchases 3 points;

Loan amount = $350,000

Points cost = 3 points × 1% × $350,000 = $10,500

Effective loan amount = $350,000 - $10,500 = $339,500

Interest rate = 4.5% / 12 = 0.375%

Number of payments=20 years × 12 = 240

Using the PMT function, we get;

PMT = $1,878.63

So, by purchasing 3 points, Marsha can save;

$1,987.13 - $1,878.63 = $108.50 per month

Over the life of the loan, which is 20 years or 240 months, the total savings would be;

$108.50 × 240 = $26,040

Therefore, Marsha would save $26,040 amount of money.

To know more about loan here

https://brainly.com/question/31292605

#SPJ1

The Maclaurin series for a function f is given by f(x)=x−x^3/3!+x^5/5!−x^7/7!+⋯+(−1)^n*x^2n+1/(2n+1)!+⋯ and converges to f(x) for all x. Let g be the function defined by g(x)=f(x2)

Answers

The Maclaurin series for g(x) is given by g(x) =[tex]x^2 - x^6/3! + x^10/5! -[/tex] [tex]x^14/7![/tex] [tex]+ ⋯ + (-1)^n*x^(4n)/(2n+1)! + ⋯[/tex]

How to the Maclaurin series of g(x)?

The function g(x) is defined as g(x) = [tex]f(x^2)[/tex], where f(x) is a function with a Maclaurin series expansion.

To find the Maclaurin series for g(x), we substitute [tex]x^2[/tex] into the Maclaurin series of f(x). The resulting series for g(x) is obtained by replacing each occurrence of x in the series for f(x) with x^2:

g(x) = [tex]f(x^2) = (x^2) - (x^2)^3/3! + (x^2)^5/5! - (x^2)^7/7! + ⋯ + (-1)^n*(x^2)^(2n+1)/(2n+1)! + ⋯[/tex]

Simplifying the terms, we have:

g(x) =[tex]x^2 - x^6/3! + x^10/5! - x^14/7! + ⋯ + (-1)^n*x^(4n+2)/(2n+1)! + ⋯[/tex]

This represents the Maclaurin series expansion for the function g(x) in terms of the original function f(x) with the argument squared.

Learn more about series

brainly.com/question/12707471

#SPJ11

Find f such that f(x) = 5/
. (16) = 49.

Answers

Let's find a function f(x) such that f(x) = 5x and f(16) = 49.

To find the function, we first plug in the given input (x = 16) and output (f(16) = 49):

49 = 5 * 16

Next, we solve for the unknown constant in the function:
49 = 80
5 = 49/80

Now, we have found the function f(x): f(x) = (49/80)x

The property that every input is related to exactly one output defines a function as a relationship between a set of inputs and a set of allowable outputs. A mapping from A to B will only be a function if every element in set A has one end and only one image in set B. Let A & B be any two non-empty sets.

Functions can also be defined as a relation "f" in which every element of set "A" is mapped to just one element of set "B." Additionally, there cannot be two pairs in a function that share the same first element.

Visit here to learn more about  function : https://brainly.com/question/12431044
#SPJ11

I need help please……..

Answers

AC is 114 because 18/114=15/95. BC is 96 because 114-18=96. BE is 3 because 3/19=18/114

Which fraction is equivalent to a whole number select all that apply? 9/3, -16/8, 7/0, -5/3, 0/5

Answers

The fraction is equivalent to a whole number  are 9/3, -16/8, 7/0, 0/5

What is a fraction?

A fraction can simply be described as the part of a whole variable, a whole numbers, or a whole element.

In mathematics, there are different types of fractions. These fractions are listed thus;

Simple fractionsProper fractionsImproper fractionsComplex fractionsMixed fractions

From the information given, we have that;

Equivalent expressions or fractions are fractions with the same solutions

Then, we have;

9/3

Divide the values

3

-16/8

-2

Learn more about fractions at: https://brainly.com/question/17220365

#SPJ1

Sam has a cylindrical storage container 7 inches tall with a radius of 5 inches.


How much cat litter will fit in the container?


Use 3. 14 for π. Round your answer to the nearest tenth.



Answer: _____________ cubic inches

Answers

The cylindrical storage container can hold 549.5 cubic inches of cat litter.

To find the volume of the cylindrical storage container, we need to use the formula:

V = πr²h

Where:

π is a constant approximately equal to 3.14

r is the radius of the container

h is the height of the container

Substituting the given values, we get:

V = 3.14 x 5² x 7

V = 549.5 cubic inches

Since we know that the  vessel can hold549.5 boxy  elevation of cat  waste, we can use this value to determine how  important cat  waste can fit in the  vessel in other units.   For  illustration, if we want to know how  numerous liters of cat  waste can fit in the  vessel, we can convert boxy  elevation to liters using the conversion factor 1 liter = 61.02 boxy  elevation.   boxy  elevation61.02 boxy  elevation per liter = 8.998 liters   thus, the spherical  storehouse  vessel can hold  roughly 9 liters of cat  waste.

Learn more about Cylinders at

https://brainly.com/question/15536278

#SPJ4

FILL IN THE BLANK. Use part I of the Fundamental Theorem of Calculus to find the derivative of f(x) = x∫4 1/1+4t⁴ dt f'(x)=________

Answers

The derivative of f(x) is: f'(x) = [tan⁻¹(2)/2] - [tan⁻¹(1/2)/2]

The Fundamental Theorem of Calculus is a pair of theorems that link the concept of differentiation and integration. It states that if a function f(x) is continuous on an interval [a, b] and F(x) is the antiderivative of f(x) on the same interval, then:

Part I: The derivative of the integral of f(x) from a to x is equal to f(x):

d/dx ∫a to x[tex]f(t) dt = f(x)[/tex]

Part II: The integral of the derivative of a function f(x) on an interval [a, b] is equal to the difference between the values of the function at the endpoints of the interval:

∫a to b [tex]f'(x) dx = f(b) - f(a)[/tex]

Using Part I of the Fundamental Theorem of Calculus, we have:

f(x) = x∫4 1/(1+4t⁴) dt

Then, by the Chain Rule, we have:

f'(x) = d/dx [x∫4 1/(1+4t⁴) dt] = ∫4 d/dx [x(1/(1+4t⁴))] dt

= ∫4 (1/(1+4t⁴)) dt

= [tan⁻¹(2t)/2]₄¹

= [tan⁻¹(2)/2] - [tan⁻¹(1/2)/2]

Therefore, the derivative of f(x) is:

f'(x) = [tan⁻¹(2)/2] - [tan⁻¹(1/2)/2]

To learn more about derivative visit: https://brainly.com/question/30365299

#SPJ11

1) Last year a computer cost $1,600 to purchase. A year later the same computer cost $1,850 to purchase. By what percent did the cost of the computer increase? (please show your work)
2)A flock of 50 geese landed at a pond. Later that day there were 75 in the pond. By what percent did the number of geese increase?

a
80%
b
50%
c
2%
d
10%

Answers

Answer:

1) 15.625% increase. 1850-1600=250. 250/1600=0.15625. 0.15625x100= 15.625.

2) 50%

Step-by-step explanation:

1) 1850-1600=250. 250/1600=0.15625. 0.15625x100= 15.625.

2) 75-50=25. 25/50=0.5. 0.5x100=50

Earthworm Rivals are building the set for

their new music video. There is a tower made

of 9 glowing bricks that stands 5. 4 meters tall. If each of the bricks is the same exact size,

how tall is each brick?

Answers

Since each of the bricks is the same exact size, then each brick is 0.6 meters tall.

To determine the height of each glowing brick, we need to divide the total height of the tower (5.4 meters) by the number of bricks (9). This gives us the average height of each brick.

Using the formula for division, we can write this as:

Height of each brick = Total height of tower / Number of bricks

Plugging in the given values, we get:

Height of each brick = 5.4 meters / 9 bricks

Simplifying this expression, we can cancel out the units of "bricks" to get:

Height of each brick = 0.6 meters

Therefore, each glowing brick in the tower is 0.6 meters tall.

Learn more about division here: https://brainly.com/question/30126004

#SPJ11

Jack starts to save at age 40 for a vacation home that he wants to buy for his 50th birthday. He will contribute $1000 each quarter to an account, which earns 2. 1% interest, compounded annually. What is the future value of this investment, rounded to the nearest dollar, when Jack is ready to purchase the vacation home?



$11,000



$11,231



$44,000



$44,924

Answers

The future value of the investment when Jack is ready to purchase the vacation home is $44,924.

To solve this problem, we can use the formula for future value of an annuity:

FV = Pmt x [(1 + r)^n - 1] / r

Where:

Pmt = $1000 (quarterly contribution)
r = 0.021 (annual interest rate)
n = 40 (number of quarters until Jack turns 50)

Plugging in the numbers, we get:

FV = $1000 x [(1 + 0.021)^40 - 1] / 0.021
FV = $44,924.38

Therefore, the future value of Jack's investment, rounded to the nearest dollar, is $44,924. So the correct answer is $44,924.

Learn more about annuity here: https://brainly.com/question/25792915

#SPJ11

Mr. Lee has a small apple orchard. There are 7 rows of tree with n trees in each row. which two expression show different ways to find the total number of trees in Mr. Lee apple orchard?

Answers

Therefore , the solution of the given problem of expressions comes out to be  7n.

What exactly is an expression?

Instead of using random estimates, it is preferable to use shifting numbers that may also prove increasing, reducing, variable or blocking. They could only help one another by trading tools, information, or solutions to issues. The justifications, components, or quantitative comments for tactics like further disagreement, production, and blending may be included in the assertion of truth equation.

Here,

By dividing the number of rows by the number of trees in each row, one can calculate the total number of trees in Mr. Lee's apple orchard. Here are two expressions that demonstrate various approaches to determining the overall number of trees:

There are 7n =  trees in all.

=> Total number of trees = (Number of rows) x (Number of trees in each row) = 7n

The total number of trees in the orchard is the outcome of both expressions.

While the second statement more directly depicts the multiplication, the first expression merely merges the two elements into a single term.

To know more about expressions visit :-

brainly.com/question/14083225

#SPJ1

Find two vectors in opposite directions that are orthogonal to the vector u. (The answers are not unique. Enter your answer as a comma-separated list of vectors.) u = (5, -4,8) Determine whether the planes are orthogonal, parallel, or neither

Answers

The cross-product of u and v:
w = u × v = (5, -4, 8) × (-8, -4, 5) = (-20, -60, 32)

Thus, w is orthogonal to u. Since we need two vectors in opposite directions, we can negate w:
-w = (20, 60, -32)
Therefore, the two orthogonal vectors in opposite directions are w = (-20, -60, 32) and -w = (20, 60, -32).

To find two vectors that are orthogonal to u, we can use the cross-product. Let v = (4,5,0) and w = (-8,0,5). Then v x u = (40,40,45) and w x u = (20,-40,20). So two vectors orthogonal to u are (40,40,45) and (20,-40,20).

To determine whether two planes are orthogonal, parallel, or neither, we can look at the normal vectors of each plane. Let the first plane be defined by the equation 2x + 3y - z = 4 and the second plane being defined by the equation :

4x + 6y - 2z = 8.

The normal vector of the first plane is (2,3,-1) and the normal vector of the second plane is (4,6,-2).

Since the dot product of these two normal vectors is -2(3) + 3(6) - 1(2) = 14, which is not equal to 0, the planes are not orthogonal.

To determine if they are parallel, we can check if the ratio of their normal vectors is constant. Dividing the second normal vector by the first, we get (4/2, 6/3, -2/-1) = (2,2,2). Since this is a constant ratio, the planes are parallel.

Learn more about Orthogonal:

brainly.com/question/31046862

#SPJ11

PLEASE HELP AND SHOW WORK!! 10 PTS IF U ANSWER

Answers

Answer:

Step-by-step explanation:

You're going to want to break up the shape into three parts, two triangles, and the rectangle.

Starting with the left-most triangle: A=(L*W)/2

The length is 4ft and the width is 3ft, multiply and divide by 2 to get:  A=6 square feet.

Do the same with the second triangle on the bottom left (L=2ft, W=2ft) to get A=2 square feet.

Now the rectangle, A=L*W and total length is 10ft (8ft+2ft) and the width is 3ft. Multiply these values to get A=30 square feet.

Last step: add up all three areas for the total area of the entire shape, 6+2+30=38.

Area= 38 square feet.

Identify the equation of the line that passes through the pair of points (0,4) and (6, −3) in slope-intercept form.

Answers

An equation of the line that passes through the pair of points (0,4) and (6, −3) in slope-intercept form is y = -7x/6 + 4.

How to determine an equation of this line?

In Mathematics and Geometry, the point-slope form of a straight line can be calculated by using the following mathematical expression:

y - y₁ = m(x - x₁)

Where:

x and y represent the data points.m represent the slope.

First of all, we would determine the slope of this line;

Slope (m) = (y₂ - y₁)/(x₂ - x₁)

Slope (m) = (-3 - 4)/(6 - 0)

Slope (m) = -7/6

At data point (0, 4) and a slope of -7/6, a linear equation for this line can be calculated by using the point-slope form as follows:

y - y₁ = m(x - x₁)

y - 4 = -7/6(x - 0)  

y = -7x/6 + 4

Read more on point-slope here: brainly.com/question/24907633

#SPJ1

Identify the line of symmetry for the function below:
g(x) = |x +9|- 11

Answers

Answer:

x = -9

Step-by-step explanation:

As this is an absolute value function, the line of symmetry is the x-value of the maximum/minimum point. An absolute value function can be denoted as y = |x - h| + k, where (h, k) is the maximum/minimum point. We only need the x-value of the maximum/minimum point, so we only have to look at "h". Now, we can use y = |x - h| + k and turn it into g(x):

y = |x - h| + k

g(x) = |x - -9| + -11   --> this means h = -9, and the line of symmetry is at x = -9

g(x) = |x + 9| - 11

Answer:

I think x equals --9

f(x)=-x^(2)-8x+19

1.whats the functions minimum value?

2.where does the minimum value occur?

Answers

The minimum value of the function is -13 and the minimum value of the function occurs at the point (4, -13).

The function F(x) is a quadratic function with a negative coefficient of the squared term.

Therefore, the function has a maximum value.

To find the maximum value, we need to find the vertex of the parabola.

The x-coordinate of the vertex is given by x = -b/2a, where a and b are the coefficients of the x² and x terms respectively.

In this case, a = -1 and b = -8, so x = -(-8)/(2(-1)) = 4.

To find the minimum value, we substitute this x-value into the function to get F(4) = -(4²) - 8(4) + 19 = -13.

Therefore, the minimum value of the function is -13.

We found in part (1) that the x-coordinate of the vertex is x = 4.

To find the y-coordinate, we substitute this x-value into the function to get F(4) = -13.

Therefore, the minimum value of the function occurs at the point (4, -13).

To learn more on Functions click:

https://brainly.com/question/30721594

#SPJ1

A buyer purchases a 2022 Jeep Grand Wagoneer Series III for $109,000 and a 2023 Cadillac Escalade V for $149,000. How much money will he spend a year for car payments?

Answers

The amount of money that he will spend a year for car payment would be = $258,000

How to calculate the total amount of money that will be spent of car payments?

To calculate the amount of money that will be used for car payment the following is carried out.

The cost for purchasing 2022 Jeep Grand Wagoneer Series III = $109,000

The cost for purchasing a 2023 Cadillac Escalade V = $149,000

The total amount spent of car payments = 109000+149000

= $258,000

Therefore, the buyer will spend up to $258,000 a year for car payment when the price for both cars he bought is being summed up.

Learn more about addition here:

https://brainly.com/question/29793687

#SPJ1

i need to figure it out

Answers

The solution to the equation 4x + 17 = 23 is x = 3/2.

How to solve the equation

It should be noted that to solve this equation, we need to isolate the variable x on one side of the equation.

First, we can subtract 17 from both sides of the equation:

4x + 17 - 17 = 23 - 17

Simplifying the left side of the equation:

4x = 6

Next, we can divide both sides of the equation by 4:

4x/4 = 6/4

Simplifying:

x = 3/2

Therefore, the solution to the equation 4x + 17 = 23 is x = 3/2.

Learn more about equations on

https://brainly.com/question/2972832

#SPJ1

A rectangular pyramid has a volume of 480 in. If a rectangular prism has a base and height congruent to the pyramid, what
is the volume of the prism? († point)

Please help!!!

Answers

After considering all the given data we come to the conclusion that the volume of the prism is 1440 in³, under the condition that A rectangular pyramid has a volume of 480 in.

The volume of a rectangular pyramid is represented by the formula
(1/3) × base area × height.
Now, the volume of a rectangular prism is given by the formula
base area × height.
Now if we consider the rectangular prism has a base and height congruent to the pyramid, then the base area of the prism is equivalent to the base area of the pyramid. Then, the volume of the prism is equivalent to three times that of the pyramid.

Hence, the volume of the pyramid is 480 in³, we can evaluate the volume of the prism
Volume of prism = 3 × Volume of pyramid
= 3 × (1/3) × Base area × Height
= Base area × Height
Then, the volume of the rectangular prism is
480 × 3
= 1440 in³.
To learn more about volume
https://brainly.com/question/27710307
#SPJ1

A square a rectangle have the same perimeter of a square has a side length of 8x units. The rectangle has a length of (5x + 12) and a width of 10 units. what will be the perimeter of both a rectangle and the square

Answers

Answer:

Step-by-step explanation:

The perimeter of a square is calculated by multiplying the length of one side by 4. Since the side length of the square is 8x units, the perimeter of the square is 4 * 8x = 32x units.

The perimeter of a rectangle is calculated by adding the lengths of all four sides or by using the formula 2 * (length + width). Since the length of the rectangle is (5x + 12) units and the width is 10 units, the perimeter of the rectangle is 2 * ((5x + 12) + 10) = 10x + 44 units.

Since both shapes have the same perimeter, we can set their perimeters equal to each other and solve for x:

32x = 10x + 44 22x = 44 x = 2

Substituting this value of x back into the expression for the perimeter of either shape, we find that the perimeter of both the square and the rectangle is 64 units.

. given that z is a standard normal random variable, a positive value of z indicates that: question 2 options: a) the standard deviation of z is negative b) the probability associated with z is negative c) the value z is to the left of the mean d) the area between zero and z is negative. e) the value z is to the right of the mean

Answers

The positive value of z indicates option e) the value z is to the right of the mean.


A standard normal random variable has a mean of 0 and a standard deviation of 1. Positive values of z represent values above the mean, while negative values of z represent values below the mean.

The probability associated with a value of z is always positive since it represents the likelihood of observing a certain value. The area between zero and z is also always positive since it represents the probability of observing a value between 0 and z.

Therefore, option e) is the correct answer as it reflects the relationship between positive values of z and their location relative to the mean.

Learn more about "z value":

https://brainly.com/question/30172182

#SPJ11

Other Questions
Below are the steps for copying Line Segment DE using dynamic geometry software. Which sequence has the steps in the correct order?1.) Draw Line Segment DE with endpoint H on the circle.2.) Construct a circle centered at the point G with radius Line Segment DE.3.) Line Segment DE Line Segment GH4.) Draw a point and label it G.~a.) 2,4,1,3~b.) 4,2,1,3~c.) 2,1,4,3~d.) 3,4,2,1 Kathleen has a fair die with 6 different-colored sides:red,yellow,blue,green,orange, and purple. She rolls the die 120 times. The die lands on the color green 18 times. Based on Kathleen's results, The experimental probability of rolling the color green is? Describe briefly the arrival of a new baby sister or brother in your house in a paragraph of 300 - 350 words How are these consistent with the rest of the star? (As we read in Chapter 6, it is insufficient to state that "our people are our biggest asset" and leave it at generalities such as that. State specifically why your people strategy is unique to your organization and why it supports other components of your organization design). How are your people practices unique to supporting the strategic agenda of your organization, and why will they give you a competitive advantage? What factors caused the correlation between the reduction in skin pigmentation in humans and migration of human populations away from the equator?*1 pointThe exposure of human populations to new diseases common to the polar regions interferred with melanin production causing lighter skin.The exposure of human populations to new sources of food common to the polar regions caused changes in physiology that resulted in the production of less melanin.The exposure of human populations to less UV light required a reduction in melanin to produce more vitamin D for healthier bones.The exposure of human populations to predators only common to the polar regions caused them to adapt by camouflaging better with the environment. Explain the key differences (not just the definitions) between the four functional areas of theorganization. Also, explain why we need all four areas for a business to prosper. In lines 840851, what are Creon and Haemon arguing about? What prejudices does Creon reveal, and what do they tell you about Creon's character? What was the name of the native american who taught the pilgrims about crops that grow well in england Joey wants to buy a $3,000 vehicle with 20 percent down for three years at 12 percent interest. What will his monthly payment be?Monthly Payment per $1,000 FinanceA. $72.82B. $89.67C. $79.70D. $119.56 A metallurgist made two purchases. The first purchase, which cost $1224, included 48 kilograms of an iron alloy and 40 kilograms of a lead alloy. The second purchase, at the same price, cost $507 and included 24 kilograms of the iron alloy and 13 kilograms of the lead alloy. Find the cost per kilogram of the iron and lead alloys. What is the cost of the iron alloy and the lead alloy? Nathan is analyzing the demand for baseball bats that illuminate when they hit something. The managing director of his company has asked him to understand the type of need that exists for such a product. What are the two types of needs that Nathan should consider?Nathan should consider the _#1?_ and _#2?_ needs that would exist for a self-illuminating baseball bat. #1fulfilledadjustedlatent#2completerealizedjustifiable Which animal has fingerprints almost identical to human. Solve 2x(x 0.5) = (x+2.5) + 5.5 by graphing. Round to the nearest thousandth.xand x Moosehead Lake in Maine is a beautiful lake, spanning 74,890 acres with a maximum depth of 246 feet. The most sought-after fish to be found in Moosehead Lake are: Landlocked salmon, lake trout, brook trout, and burbot. According to Tim Obrey, a regional fisheries biologist with the Maine Department of Inland Fisheries and Wildlife, the fishing at Moosehead declined from phenomenal to just "really good" during the late 1980s, but has rebounded due to careful management. This table shows the populations of three major fish in Moosehead Lake over a 30 year period. Moosehead Lake was illegally stocked with smallmouth bass in 1970. What was the effect on the populations of native salmon and trout? Express all real numbers greater than 2 in interval notation Andre owns a condominium with a value of $155,000. He has a stock portfolio worth $8,100. He owes $3,300 on his car, which is valued at $9,100. He has $7,600 in student loans to repay. He has a credit card balance of $4,327. He also has $2,600 in a bank account. Construct a net worth statement to find Andre's net worth. Complete the following sentences:Wave speed is NOT dependent on _________ or ____________. Instead, wave speed only changes when the ______________ changes. This means that if a sound wave is traveling in pure water and the frequency doubles, the wavelength must _________. You need to prepare an acetate buffer of pH 5. 17 from a 0. 660 M acetic acid solution and a 2. 63 M KOH solution. If you have 930 mL of the acetic acid solution, how many milliliters of the KOH solution do you need to add to make a buffer of pH 5. 17 ? The pa of acetic acid is 4. 76. Be sure to use appropriate significant figures The money in Maya's college savings account earns 2 1/5% interest. Which value is less than 2 1/5%?A. 0. 0215B. 11/5C. 0. 022D. 11/500 what term is used to describe a pacemaker's ability to successfully stimulate a myocardial response?